8
$\begingroup$

Do homotopy colimits commute with homotopy colimits? The setting I am thinking of is that of a triangulated category with a model, but it would be interesting to have more general answers as well. A good reference would also be appreciated.

$\endgroup$

1 Answer 1

8
$\begingroup$

Yes. See theorem 24.9 of this paper. Here's a somewhat less abstract reference.

$\endgroup$
1
  • 2
    $\begingroup$ Ah, it was a few years ago when the answer was posted, but it turns out this is exactly what I was looking for right now. Thank you! For those who want to use this in their papers, let me just leave a remark here that it was published as part of Memoir of AMS, Vol 155, No. 736 (2002). $\endgroup$ Mar 2, 2012 at 9:34

Your Answer

By clicking “Post Your Answer”, you agree to our terms of service and acknowledge you have read our privacy policy.